LSAT and Law School Admissions Forum

Get expert LSAT preparation and law school admissions advice from PowerScore Test Preparation.

 reop6780
  • Posts: 265
  • Joined: Jul 27, 2013
|
#13278
This is justify type.

I thought I was solving assumption problem when I was timing.

I was between answer B and E. ( I got it wrong by choosing E)

Is there big difference by setting this justify problem as assumption?

Also, they both belong to the 2nd family which allows to bring information outside stimuli.

How is answer E different from answer B, and what makes B wrong?
 Adam Tyson
PowerScore Staff
  • PowerScore Staff
  • Posts: 5153
  • Joined: Apr 14, 2011
|
#13282
Good questions - let's start with the most basic, which is about identifying the question type. The stem gives you two clues that this is NOT an assumption question, but a justify question. First, it asks about the conclusion being properly drawn - another way of saying the conclusion must be true, and that's a justify idea. Second, and this is very important - the stem uses the word "if" ("If which one of the following were assumed"). Assumption questions almost never (maybe never?) use the word "if" - they ask more directly which answer must be assumed.

That said, if you should confuse this question for an assumption, your prephrase would probably look a lot like answer B. The auther clearly assumed that there is some direct link between the number of words for colors and the ability to distinguish colors from one another. The assumption wouldn't be as strong as B, though - it's more than we need. All we need is something like "the fewer the number of words for colors, the fewer colors one can distinguish" or "there is a direct correlation between the number of words for colors in a language and the number of colors speakers of that language can distinguish."

E doesn't tie the idea of perception of colors to the number of words in the language. Instead, it distracts us with interesting info about flora and fauna that really proves nothing about what colors the speaker can distinguish. Just because the plants and animals are all green and brown doesn't mean the people there can't distinguish mauve from puce, right?

Hope that helped!
 zubinator3000
  • Posts: 4
  • Joined: Nov 15, 2017
|
#43796
Can someone please demonstrate how the Mechanistic Approach would be used for this problem.

Much appreciated!
 Adam Tyson
PowerScore Staff
  • PowerScore Staff
  • Posts: 5153
  • Joined: Apr 14, 2011
|
#43798
You bet, zubinator! The conclusion about colors brings up the concept of "ability to distinguish", and that is a new concept and must be addressed by the correct answer. The premises talk about the number of words, so I want to link numbers of words to ability to distinguish what those words represent. Only answer B makes that link, and boom, we're done!
 whardy21
  • Posts: 48
  • Joined: Sep 30, 2018
|
#64474
I chose E. Can someone please explain how B is the correct answer. What does sensory quality have to do with this question? Please explain. Thanks
User avatar
 Dave Killoran
PowerScore Staff
  • PowerScore Staff
  • Posts: 5853
  • Joined: Mar 25, 2011
|
#64476
whardy21 wrote:I chose E. Can someone please explain how B is the correct answer. What does sensory quality have to do with this question? Please explain. Thanks
Hi W,

The sensory quality reference here is related to their discussion of colors in the stimulus, as in you use one of your senses to tell colors apart. So, that is an umbrella term they use to throw you off but which definitely relates to the topic at hand. They've been doing this a lot recently to throw test takers off, so this is a great example to examine.

Then, as Adam so nicely put it above, "The conclusion about colors brings up the concept of "ability to distinguish", and that is a new concept and must be addressed by the correct answer. The premises talk about the number of words, so I want to link numbers of words to ability to distinguish what those words represent. Only answer B makes that link, and boom, we're done!"

Thanks!
 MeliXi
  • Posts: 19
  • Joined: Dec 12, 2020
|
#83445
I feel like I didn't really understand this question. I read the stimulus & couldn't prephase anything & was unable to distinguish what part of the argument was missing. I didn't really have any idea what the correct answer was so I just chose E.
so how do I go about this question & what's the proper step-by-step thought process for it?
 tetsuya0129
  • Posts: 73
  • Joined: Jun 20, 2018
|
#83652
Hi Powerscore staff,

I wanna know whether my approach is correct.

For me, the stimulus presents a conditional argument by its last sentence ("Therefore,..."), because it uses "must". And, (B) presents a bi-conditional statement.

Any help or comments would be greatly appreciated.

Thanks,
Leon
 Adam Tyson
PowerScore Staff
  • PowerScore Staff
  • Posts: 5153
  • Joined: Apr 14, 2011
|
#83756
MeliXi: Start by focusing on the conclusion. What is the author trying to prove? The use of "therefore" should clue you in there - the conclusion is that people whose languages have fewer words for colors must perceive fewer colors. See the gap between those two ideas? Who says perception has anything to do with language? Maybe I can perceive 11 different shades of blue, but to me they are all just "blue"? So to justify this rather odd, very strong conclusion, I need to know that languages have words for every variation of color that the speakers can perceive.

Leon: Many conclusions have a statement of certainty, like a "must" statement, that are not conditional arguments. An argument is conditional only if it is based on "if this, then that" claims. Consider this non-conditional argument: "Denver has a lot in common with Portland, so they must have very similar weather." A terrible argument based on a false analogy, but not at all conditional, despite the "must" in the conclusion.

Nonetheless, most Justify the Conclusion questions do introduce a conditional relationship, even when the argument in the stimulus is not conditional, in the sense that they will all boil down to "if the premises are true, then the conclusion must be true." Most of the answer choices to this question are conditional statements, so that is not enough, by itself, to help select a correct answer. Instead, we need to find the right conditional statement, which is the one that connects the numbers of words for something to the number of those things perceived. That's where answer B wins!
 tetsuya0129
  • Posts: 73
  • Joined: Jun 20, 2018
|
#83787
Thank you Adam!

Get the most out of your LSAT Prep Plus subscription.

Analyze and track your performance with our Testing and Analytics Package.